Anda di halaman 1dari 48

No part of this publication may be reproduced or distributed in any form or any means, electronic, mechanical,

photocopying, or otherwise without the prior permission of the author.

GATE SOLVED PAPER


Electrical Engineering
2008

Copyright © By NODIA & COMPANY

Information contained in this book has been obtained by authors, from sources believes to be reliable. However,
neither Nodia nor its authors guarantee the accuracy or completeness of any information herein, and Nodia nor its
authors shall be responsible for any error, omissions, or damages arising out of use of this information. This book
is published with the understanding that Nodia and its authors are supplying information but are not attempting
to render engineering or other professional services.

NODIA AND COMPANY


B-8, Dhanshree Tower Ist, Central Spine, Vidyadhar Nagar, Jaipur 302039
Ph : +91 - 141 - 2101150
www.nodia.co.in
email : enquiry@nodia.co.in
GATE SOLVED PAPER - EE
2008

Q.1 - Q.20 carry one mark each.


Q. 1 The number of chords in the graph of the given circuit will be

(A) 3
.
(B) 4
i n
(C) 5
o
(D) 6

c
Sol. 1 No. of chords is given as

a .
i
l = b-n+1

d
b " no. of branches

o
n " no. of nodes

.n
l " no. of chords
b = 6, n = 4

w
l = 6 - 4 + 1= 3

w
Hence (A) is correct option.

w
Q. 2 The Thevenin’s equivalent of a circuit operation at w = 5 rads/s, has
Voc = 3.71+ - 15.9% V and Z0 = 2.38 - j0.667 W . At this frequency, the minimal

©
realization of the Thevenin’s impedance will have a
(A) resistor and a capacitor and an inductor
(B) resistor and a capacitor
(C) resistor and an inductor
(D) capacitor and an inductor
Sol. 2 Hence (A) is correct option.
Impedance Zo = 2.38 - j0.667 W
Constant term in impedance indicates that there is a resistance in the circuit.
Assume that only a resistance and capacitor are in the circuit, phase difference
in thevenin voltage is given as
q =- tan- 1 (wCR) (Due to capacitor)
j
Zo = R -
wC
So, 1 = 0.667
wC
and R = 2.38 W
q =- tan- 1 b 1 # 2.38 l
0.667
GATE SOLVED PAPER - EE 2008

=- 74.34c =-
[ 15.9c
given Voc = 3.71+ - 15.9c
So, there is an inductor also connected in the circuit
Q. 3 A signal e - at sin (wt) is the input to a real Linear Time Invariant system. Given K
and f are constants, the output of the system will be of the form Ke - bt sin (vt + f)
where
(A) b need not be equal to a but v equal to w
(B) v need not be equal to w but b equal to a
(C) b equal to a and v equal to w
(D) b need not be equal to a and v need not be equal to w
Sol. 3 Hence (D) is correct option.
L
Let x (t) X (s)

in
L
y (t) Y (s)

.
L
h (t) H (s)

co
So output of the system is given as

a.
Y (s) = X (s) H (s)
e - st X (s)

i
x (t - t)
L
Now for input (shifting property)
- st

d
h (t - t)
L
e H (s)

o
So now output is Y' (s) = e - st X (s) $ e - ts H (s)

n
Y' (s) = e - 2st X (s) H (s)

. Y' (s) = e - 2st Y (s)


Or
w y' (t) = y (t - 2t)
Q. 4
w
X is a uniformly distributed random variable that takes values between 0 and 1.

w
The value of E {X3} will be
(A) 0 (B) 1/8

Sol. 4
©
(C) 1/4 (D) 1/2
X is uniformly distributed between 0 and 1
So probability density function
1, 0 < x < 1
fX (X) = )
0, otherwise
So,
1
E {X3} = #0 X3 fX (X) dx
1
= #0 X3 (1) dx
4 1
= :X D
4 0
=1
4
Hence (C) is correct option
Q. 5 The characteristic equation of a (3 # 3 ) matrix P is defined as
a (l) = lI - P = l3 + l2 + 2l + 1 = 0
If I denotes identity matrix, then the inverse of matrix P will be
(A) (P2 + P + 2I) (B) (P2 + P + I)
(C) - (P2 + P + I) (D) - (P2 + P + 2I)
GATE SOLVED PAPER - EE 2008

Sol. 5 According to CAYLEY-HAMILTON Theorem every non-singular square matrix


satisfies its own characteristic equation.
Characteristic equation
a (l) = lI - P = l3 + l2 + 2l + 1 = 0
Matrix P satisfies above equation
P 3 + P 2 + 2P + I = 0
I =- (P3 + P2 + 2P)
Multiply both sides by P- 1
P- 1 =- (P2 + P + 2I)
Hence (D) is correct option.
Q. 6 If the rank of a (5 # 6) matrix Q is 4, then which one of the following statement
is correct ?
(A) Q will have four linearly independent rows and four linearly independent
columns
(B) Q will have four linearly independent rows and five linearly independent

in
columns

.
T
(C) QQ will be invertible

co
(D) QT Q will be invertible

.
Sol. 6 Rank of a matrix is no. of linearly independent rows and columns of the matrix.

i a
Here Rank r (Q) = 4

d
So Q will have 4 linearly independent rows and flour independent columns.

o
Hence (A) is correct option

.n
Q. 7 A function y (t) satisfies the following differential equation :
dy (t)

w
+ y (t) = d (t)
dt

w
where d (t) is the delta function. Assuming zero initial condition, and denoting the
unit step function by u (t), y (t) can be of the form
(A) et
w (B) e - t

©
(C) et u (t) (D) e - t u (t)
Sol. 7 Given differential equation for the function
dy (t)
+ y (t) = d (t)
dt
Taking Laplace on both the sides we have,
sY (s) + Y (s) = 1
(s + 1) Y (s) = 1
Y (s) = 1
s+1
Taking inverse Laplace of Y (s)
y (t) = e- t u (t), t > 0
Hence (D) is correct option.
Q. 8 The equivalent circuits of a diode, during forward biased and reverse biased
conditions, are shown in the figure.
(I)
GATE SOLVED PAPER - EE 2008

(II)

If such a diode is used in clipper circuit of figure given above, the output voltage
V0 of the circuit will be

. in
. co
i a
o d
. n
w
w
w
Sol. 8
©
Assume the diode is in reverse bias so equivalent circuit is

Output voltage V0 = 10 sin wt # 10 = 5 sin wt


10 + 10
Due to resistor divider, voltage across diode VD < 0 (always). So it in reverse bias
for given input.
Output, V0 = 5 sin wt
Hence (A) is correct option.
Q. 9 Two 8-bit ADCs, one of single slope integrating type and other of successive
approximate type, take TA and TB times to convert 5 V analog input signal to
equivalent digital output. If the input analog signal is reduced to 2.5 V, the
approximate time taken by the two ADCs will respectively, be
(A) TA, TB (B) TA /2, TB
(C) TA, TB /2 (D) TA /2, TB /2
GATE SOLVED PAPER - EE 2008

Sol. 9 Conversion time does not depend on input voltage so it remains same for both
type of ADCs.
Hence (A) is correct option
Q. 10 An input device is interfaced with Intel 8085A microprocessor as memory mapped
I/O. The address of the device is 2500H. In order to input data from the device
to accumulator, the sequence of instructions will be
(A) LXI H, 2500H (B) LXI H, 2500H
MOV A, M MOV M, A
(C) LHLD 2500H (D) LHLD 2500H
MOV A, M MOV M, A
Sol. 10 Hence ( ) is Correct Option
Q. 11 Distributed winding and short chording employed in AC machines will result in
(A) increase in emf and reduction in harmonics
(B) reduction in emf and increase in harmonics

in
(C) increase in both emf and harmonics

.
(D) reduction in both emf and harmonics

co
Sol. 11 Distributed winding and short chording employed in AC machine will result in

.
reduction of emf and harmonics.

i a
Hence (D) is correct option.

d
Q. 12 Three single-phase transformer are connected to form a 3-phase transformer bank.

o
The transformers are connected in the following manner :

.n
w
w
w
©
The transformer connecting will be represented by
(A) Y d0 (B) Y d1
(C) Y d6 (D) Y d11
Sol. 12 Transformer connection will be represented by Y d1.
Hence (B) is correct option.
Q. 13 In a stepper motor, the detent torque means
(A) minimum of the static torque with the phase winding excited
(B) maximum of the static torque with the phase winding excited
(C) minimum of the static torque with the phase winding unexcited
(D) maximum of the static torque with the phase winding unexcited
Sol. 13 Detent torque/Restraining toque:
The residual magnetism in the permanent magnetic material produced.
The detent torque is defined as the maximum load torque that can be applied to
the shaft of an unexcited motor without causing continuous rotation. In case the
motor is unexcited.
Hence (D) is correct option.
GATE SOLVED PAPER - EE 2008

Q. 14 A two machine power system is shown below. The Transmission line XY has
positive sequence impedance of Z1 W and zero sequence impedance of Z0 W

An ‘a’ phase to ground fault with zero fault impedance occurs at the centre of the
transmission line. Bus voltage at X and line current from X to F for the phase
‘a’, are given by Va Volts and Ia amperes, respectively. Then, the impedance
measured by the ground distance relay located at the terminal X of line XY will
be given by
(A) ^Z1 /2h W (B) ^Z0 /2h W
(C) (Z0 + Z1) /2 W (D) ^Va /Ia h W

in
Sol. 14 Given for X to F section of phase ‘a’

.
Va -Phase voltage and Ia -phase current.

co
Impedance measured by ground distance,
Bus voltage

.
Relay at X =
Current from phase 'a'
= Va W
Ia
i a
Hence (D) is correct option.

o d
n
Q. 15 An extra high voltage transmission line of length 300 km can be approximate by

.
a lossless line having propagation constant b = 0.00127 radians per km. Then the

w
percentage ratio of line length to wavelength will be given by
(A) 24.24 % (B) 12.12 %
(C) 19.05 %
w (D) 6.06 %
Sol. 15
w
For EHV line given data is

©
Length of line = 300 km and b = 0.00127 S rad/km
wavelength l = 2p = 2p = 4947.39 km
b 0.00127
So l %= 300 100 = 0.06063 # 100
l 4947.39 #
l % = 6.063
l
Hence (D) is correct option.
Q. 16 A-3-phase transmission line is shown in figure :

Voltage drop across the transmission line is given by the following equation :
R V R VR V
S3 Va W SZs Zm Zm WSIa W
S3 Vb W = SZm Zs Zm WSIb W
SS3 V WW SSZ Z Zs WWSSIc WW
c m m
T X T XT X
GATE SOLVED PAPER - EE 2008

Shunt capacitance of the line can be neglect. If the has positive sequence impedance
of 15 W and zero sequence impedance of 48 W, then the values of Zs and Zm will
be
(A) Zs = 31.5 W; Zm = 16.5 W
(B) Zs = 26 W; Zm = 11 W
(C) Zs = 16.5 W; Zm = 31.5 W
(D) Zs = 11 W; Zm = 26 W
Sol. 16 For three phase transmission line by solving the given equation
RDV V R(X - X ) 0 0 VRI V
S aW S s m WS aW
We get, SDVbW = S 0 (Xs - Xm) 0 WSIbW
SSDV WW SS 0 0 (Xs + 2Xm)WWSSIcWW
c
T X T XT X
Zero sequence Impedance = Xs + 2Xm = 48 ...(1)
and Positive Sequence Impedance = Negative Sequence Impedance
= (Xs - Xm)
= 15 ...(2)

in
By solving equation (1) and (2)

.
Zs or Xs = 26 and Zm or Xm = 11

co
Hence (B) is correct option.
Q. 17

a .
In the single phase voltage controller circuit shown in the figure, for what range

i
of triggering angle (a), the input voltage (V0) is not controllable ?

o d
.n
w
w
(A) 0c < a < 45c (B) 45c < a < 135c

w
(C) 90c < a < 180c (D) 135c < a < 180c
Sol. 17

©
Hence (A) is correct option.

R + jXL = 50 + 50j
` tan f = wL = 50 = 1
R 50
f = 45c
so, firing angle ‘a’ must be higher the 45c, Thus for 0 < a < 45c, V0 is
uncontrollable.
Q. 18 A 3-phase voltage source inverter is operated in 180c conduction mode. Which
one of the following statements is true ?
(A) Both pole-voltage and line-voltage will have 3rd harmonic components
GATE SOLVED PAPER - EE 2008

(B) Pole-voltage will have 3rd harmonic component but line-voltage will be free
from 3rd harmonic
(C) Line-voltage will have 3rd harmonic component but pole-voltage will be free
from 3rd harmonic
(D) Both pole-voltage and line-voltage will be free from 3rd harmonic
components
Sol. 18 A 3-f voltage source inverter is operated in 180c mode in that case third harmonics
are absent in pole voltage and line voltage due to the factor cos (np/6). so both
are free from 3rd harmonic components.
Hence (D) is correct option.
Q. 19 The impulse response of a causal linear time-invariant system is given as h (t).
Now consider the following two statements :

in
Statement (I): Principle of superposition holds

.
Statement (II): h (t) = 0 for t < 0

co
Which one of the following statements is correct ?
(A) Statements (I) is correct and statement (II) is wrong

.
(B) Statements (II) is correct and statement (I) is wrong

a
i
(C) Both Statement (I) and Statement (II) are wrong

d
(D) Both Statement (I) and Statement (II) are correct
Sol. 19

n o
Since the given system is LTI, So principal of Superposition holds due to linearity.

.
For causal system h (t) = 0 , t < 0

w
Both statement are correct.

w
Hence (D) is correct option.

Q. 20
w
It is desired to measure parameters of 230 V/115 V, 2 kVA,

©
single-phase transformer. The following wattmeters are available in a laboratory:
W1 : 250 V, 10 A, Low Power Factor
W2 : 250 V, 5 A, Low Power Factor
W3 : 150 V, 10 A, High Power Factor
W4 : 150 V, 5 A, High Power Factor

The Wattmeters used in open circuit test and short circuit test of the transformer
will respectively be
(A) W1 and W2 (B) W2 and W4
(C) W1 and W4 (D) W2 and W3
Sol. 20 Given: 1-f transformer, 230 V/115 V, 2 kVA
W1 : 250 V, 10 A, Low Power Factor
W2 : 250 V, 5 A, Low Power Factor
W3 : 150 V, 10 A, High Power Factor
W4 : 150 V, 5 A, High Power Factor
In one circuit test the wattmeter W2 is used and in short circuit test of transformer
W3 is used.
Hence (D) is correct option.
GATE SOLVED PAPER - EE 2008

Q.21 to Q.75 carry two marks each.


Q. 21 The time constant for the given circuit will be

(A) 1/9 s (B) 1/4 s


(C) 4 s (D) 9 s
Sol. 21 Time constant of the circuit can be calculated by simplifying the circuit as follows

. in
. co
i a
o d
.n
Ceq = 2 F
3

w
Equivalent Resistance

w
w
© Req = 3 + 3 = 6 W
Time constant t = Req Ceq
= 6 # 2 = 4 sec
3
Hence (C) is correct option
Q. 22 The resonant frequency for the given circuit will be

(A) 1 rad/s (B) 2 rad/s


(C) 3 rad/s (D) 4 rad/s
Sol. 22 Impedance of the circuit is
GATE SOLVED PAPER - EE 2008

1
R
jwC
Z = jw L + 1
j wC +R
R 1 - jwCR
= jw L +
1 + jwCR # 1 - jwCR
R (1 - jwCR)
= jw L +
1 + w2 C2 R2
jwL (1 + w2 C2 R2) + R - jwCR2
=
1 + w2 C2 R2
R j [wL (1 + w2 C2 R2) - wCR2]
= 2 2 2 +
1+w C R 1 + w2 C2 R2

For resonance Im (Z) = 0


So, wL (1 + w2 C2 R2) = wCR2
L = 0.1 H, C = 1 F, R = 1 W

in
So, w # 0.1 [1 + w2 (1) (1)] = w (1) (1) 2
1 + w2 = 10
.
co
& w= 9 = 3 rad/sec
Hence (C) is correct option.

a .
i
Q. 23 Assuming ideal elements in the circuit shown below, the voltage Vab will be

o d
. n
w
w
w
(A) - 3 V (B) 0 V
(C) 3 V (D) 5 V
Sol. 23
©
By applying KVL in the circuit
Vab - 2i + 5 = 0
i = 1 A,
Vab = 2 # 1 - 5 =- 3 Volt
Hence (A) is correct option
Q. 24 A capacitor consists of two metal plates each 500 # 500 mm2 and spaced 6 mm
apart. The space between the metal plates is filled with a glass plate of 4 mm
thickness and a layer of paper of 2 mm thickness. The relative primitivities of
the glass and paper are 8 and 2 respectively. Neglecting the fringing effect, the
capacitance will be (Given that e0 = 8.85 # 10 - 12 F/m )
(A) 983.3 pF (B) 1475 pF
(C) 637.7 pF (D) 9956.25 pF
Sol. 24 Here two capacitance C1 and C2 are connected in series, so equivalent capacitance
is
Ceq = C1 C2
C1 + C 2
GATE SOLVED PAPER - EE 2008

C1 = e0 er1 A
d1
- 12 -6
= 8.85 # 10 # 8 # 500 -3
# 500 # 10
4 # 10
- 11
= 442.5 # 10 F

C2 = e0 er2 A
d2
- 12 -6
= 8.85 # 10 # 2 # 500 -3
# 500 # 10
2 # 10
- 11
= 221.25 # 10 F
- 11 - 11
Ceq = 442.5 # 10 - 11 # 221.25 # 10- 11

in
442.5 # 10 + 221.25 # 10

.
= 147.6 # 10- 11

co
- 1476 pF
Hence (B) is correct option.
Q. 25

a .
A coil of 300 turns is wound on a non-magnetic core having a mean circumference

d i
of 300 mm and a cross-sectional area of 300 mm2. The inductance of the coil
corresponding to a magnetizing current of 3 A will be

o
(Given that m0 = 4p # 10 - 7 H/m)

.n
(A) 37.68 mH (B) 113.04 mH

w
(C) 3.768 mH (D) 1.1304 mH

w
Sol. 25 Hence (B) is correct option.
Circumference l = 300 mm

w no. of turns n = 300

©
Cross sectional area A = 300 mm2
m n2 A
Inductance of coil L = 0
l
4p # 10- 7 # (300) 2 # 300 # 10- 6
=
(300 # 10- 3)
= 113.04 mH
Q. 26 In the circuit shown in the figure, the value of the current i will be given by

(A) 0.31 A (B) 1.25 A


(C) 1.75 A (D) 2.5 A
GATE SOLVED PAPER - EE 2008

Sol. 26 By writing node equations at node A and B


Va - 5 + Va - 0 = 0
1 1

2Va - 5 = 0
& Va = 2.5 V
Similarly
Vb - 4Vab ++Vb - 0 = 0
3 1
Vb - 4 (Va - Vb)
+ Vb = 0
3
Vb - 4 (2.5 - Vb) + 3Vb = 0
8Vb - 10 = 0

in
& Vb = 1.25 V
Current
.
i = Vb = 1.25 A

co
1

.
Hence (B) is correct option.

a
Two point charges Q1 = 10 mC and Q2 = 20 mC are placed at coordinates (1,1,0)

i
Q. 27

and (- 1, - 1, 0) respectively. The total electric flux passing through a plane


z = 20 will be
(A) 7.5 mC
o d (B) 13.5 mC
(C) 15.0 mC
. n (D) 22.5 mC

w
Sol. 27 Hence ( ) is Correct Option

w
Q. 28 Given a sequence x [n], to generate the sequence y [n] = x [3 - 4n], which one of the

w
following procedures would be correct ?
(A) First delay x (n) by 3 samples to generate z1 [n], then pick every 4th sample

©
of z1 [n] to generate z2 [n], and than finally time reverse z2 [n] to obtain y [n].
(B) First advance x [n] by 3 samples to generate z1 [n], then pick every 4th sample
of z1 [n] to generate z2 [n], and then finally time reverse z2 [n] to obtain y [n]
(C) First pick every fourth sample of x [n] to generate v1 [n], time-reverse v1 [n] to
obtain v2 [n], and finally advance v2 [n] by 3 samples to obtain y [n]
(D) First pick every fourth sample of x [n] to generate v1 [n], time-reverse v1 [n] to
obtain v2 [n], and finally delay v2 [n] by 3 samples to obtain y [n]
Sol. 28 In option (A)
z1 [n] = x [n - 3]
z2 [n] = z1 [4n] = x [4n - 3]
y [n] = z2 [- n] = x [- 4n - 3] =
Y x [3 - 4n]
In option (B)
z1 [n] = x [n + 3]
z2 [n] = z1 [4n] = x [4n + 3]
y [n] = z2 [- n] = x [- 4n + 3]
In option (C)
v1 [n] = x [4n]
GATE SOLVED PAPER - EE 2008

v2 [n] = v1 [- n] = x [- 4n]
y [n] = v2 [n + 3] = x [- 4 (n + 3)] =
Y x [3 - 4n]
In option (D)
v1 [n] = x [4n]
v2 [n] = v1 [- n] = x [- 4n]
y [n] = v2 [n - 3] = x [- 4 (n - 3)] =
Y x [3 - 4n]
Hence (B) is correct option.
Q. 29 A system with x (t) and output y (t) is defined by the input-output relation :
- 2t
y (t) = # x (t) dt
-3
The system will be
(A) Casual, time-invariant and unstable
(B) Casual, time-invariant and stable
(C) non-casual, time-invariant and unstable
(D) non-casual, time-variant and unstable

in
Sol. 29 Input-output relation

.
- 2t
#- 3x (t) dt

co
y (t) =

.
Causality :

a
Since y (t) depends on x (- 2t), So it is non-causal.

i
Time-variance :

d
- 2t
y (t) = #- 3x (t - t0) dt =Y y (t - t0)
o
.n
So this is time-variant.
Stability :

w
Output y (t) is unbounded for an bounded input.
For example
Let
wx (t) = e - t (bounded)

w
- 2t - t - 2t
y (t) = # e - t dt = 8 e B $ Unbounded
-3 - 1 -3

©
Hence (D) is correct option.
A signal x (t) = sinc (at) where a is a real constant ^sinc (x) = px h is the input
sin (px)
Q. 30
to a Linear Time Invariant system whose impulse response h (t) = sinc (bt), where
b is a real constant. If min (a, b) denotes the minimum of a and b and similarly,
max (a, b) denotes the maximum of a and b, and K is a constant, which one of
the following statements is true about the output of the system ?
(A) It will be of the form Ksinc (gt) where g = min (a, b)
(B) It will be of the form Ksinc (gt) where g = max (a, b)
(C) It will be of the form Ksinc (at)
(D) It can not be a sinc type of signal
Sol. 30 Output y (t) of the given system is
y (t) = x (t) ) h (t)
Or Y (jw) = X (jw) H (jw)
given that
x (t) = sinc (at)
h (t) = sinc (bt)
Fourier transform of x (t) and h (t) are
GATE SOLVED PAPER - EE 2008

X (jw) = F [x (t)] = p rect` w j, - a < w < a


a 2a
H (jw) = F [h (t)] = rect` w j, - b < w < b
p
b 2b
2
Y (jw) = p rect` j rect` w j
w
ab 2a 2b

So, Y (jw) = K rect ` w j

in
2g

.
Where g = min (a, b)

co
And y (t) = K sinc (gt)
Hence (A) is correct option.
Q. 31

a .
Let x (t) be a periodic signal with time period T , Let y (t) = x (t - t0) + x (t + t0)

i
for some t0 . The Fourier Series coefficients of y (t) are denoted by bk . If bk = 0 for

d
all odd k , then t0 can be equal to

o
(A) T/8 (B) T/4

. n
(C) T/2 (D) 2T

w
Sol. 31 Let ak is the Fourier series coefficient of signal x (t)
Given y (t) = x (t - t0) + x (t + t0)

w
Fourier series coefficient of y (t)

w
bk = e - jkwt ak + e jkwt ak
0 0

bk = 2ak cos kwt0

©
bk = 0 (for all odd k )
kwt0 = p , k " odd
2
k 2 p t = p
T 0 2
For k = 1, t0 = T
4
Hence (B) is correct option.
Q. 32 H (z) is a transfer function of a real system. When a signal x [n] = (1 + j) n is the
input to such a system, the output is zero. Further, the Region of convergence
(ROC) of ^1 - 12 z - 1h H(z) is the entire Z-plane (except z = 0 ). It can then be
inferred that H (z) can have a minimum of
(A) one pole and one zero (B) one pole and two zeros
(C) two poles and one zero (D) two poles and two zeros
Sol. 32 Hence ( ) is correct option.

Q. 33 Given X (z) = z with z > a , the residue of X (z) zn - 1 at z = a


(z - a) 2
for n $ 0 will be
(A) an - 1 (B) an
(C) nan (D) nan - 1
GATE SOLVED PAPER - EE 2008

Sol. 33 Hence (D) is correct option.


Given that X (z) = z , z >a
(z - a) 2
Residue of X (z) zn - 1 at z = a is
= d (z - a) 2 X (z) zn - 1 z = a
dz
= d (z - a) 2 z zn - 1
dz (z - a) 2 z=a
= d zn
dz z = a
= nzn - 1 z = a = nan - 1
Q. 34 Consider function f (x) = (x2 - 4) 2 where x is a real number. Then the function
has
(A) only one minimum (B) only tow minima
(C) three minima (D) three maxima
Sol. 34 Given function
f (x) = (x2 - 4) 2

in
f' (x) = 2 (x2 - 4) 2x
To obtain minima and maxima
.
co
f' (x) = 0

.
4x (x2 - 4) = 0

a
x = 0, x2 - 4 = 0 & x = ! 2
So, x = 0, + 2, - 2

d i
f'' (x) = 4x (2x) + 4 (x2 - 4)
= 12x2 - 16
o
.n
x = 0, f'' (0) = 12 (0) 2 - 16 =- 16 < 0

w
For (Maxima)
2
x =+ 2, f'' (2) = 12 (2) - 16 = 32 > 0 (Minima)

w
x =- 2, f'' (- 2) = 12 (- 2) 2 - 16 = 32 > 0 (Minima)

w
So f (x) has only two minima
Hence (B) is correct option.
Q. 35
©
Equation ex - 1 = 0 is required to be solved using Newton’s method with an
initial guess x0 =- 1. Then, after one step of Newton’s method, estimate x1 of the
solution will be given by
(A) 0.71828 (B) 0.36784
(C) 0.20587 (D) 0.00000
Sol. 35 An iterative sequence in Newton-Raphson method can obtain by following
expression
f (xn)
xn + 1 = xn -
f' (xn)
We have to calculate x1 , so n = 0
f (x 0)
x1 = x 0 - , Given x 0 =- 1
f' (x 0)
f (x 0) = ex - 1 = e- 1 - 1
0

=- 0.63212
f' (x 0) = ex = e- 1
0

= 0.36787
GATE SOLVED PAPER - EE 2008

(- 0.63212)
So, x1 =- 1 -
(0.36787)
=- 1 + 1.71832
= 0.71832
Hence (A) is correct option
Q. 36 A is m # n full rank matrix with m > n and I is identity matrix. Let matrix
A' = (AT A) - 1 AT , Then, which one of the following statement is FALSE ?
(A) AA'A = A (B) (AA') 2
(C) A'A = I (D) AA'A = A'
Sol. 36 Hence (D) is correct option
A' = (AT A) - 1 AT
= A- 1 (AT ) - 1 AT

in
= A- 1 I

.
-1
Put A' = A I in all option.

co
option (A) AA'A = A
AA- 1 A = A

a .
i
A = A (true)

d
2
option (B) (AA') = I

o
(AA- 1 I) 2 = I

. n
2
(I) = I (true)

w
option (C) A'A = I

w
A- 1 IA = I

w
I = I (true)
option (D) AA'A = A'

© AA- 1 IA = A =
Y A' (false)

Q. 37 A differential equation dx/dt = e - 2t u (t), has to be solved using trapezoidal rule


of integration with a step size h = 0.01 s. Function u (t) indicates a unit step
function. If x (0 -) = 0 , then value of x at t = 0.01 s will be given by
(A) 0.00099 (B) 0.00495
(C) 0.0099 (D) 0.0198
Sol. 37 Hence (C) is correct option
dx = e- 2t u (t)
dt
x = # e- 2t u (t) dt
1
x = # e- 2t dt
0
1
x = # f (t) dt ,
0
t = .01 s
From trapezoid rule
f (t) dt = h 6f (0) + f (.01)@
t 0 + nh
#
t0 2
GATE SOLVED PAPER - EE 2008

2 6
#0 f (t) dt = .01 e0 + e- .02@, h = .01
1

= .0099
Q. 38 Let P be a 2 # 2 real orthogonal matrix and x is a real vector [x1, x2] T with length
x = (x12 + x22) 1/2 . Then, which one of the following statements is correct ?
(A) Px # x where at least one vector satisfies Px < x
(B) Px # x for all vector x
(C) Px $ x where at least one vector satisfies Px > x
(D) No relationship can be established between x and Px
Sol. 38 P is an orthogonal matrix So PPT = I
cos q - sin q
P =>
cos q H
Let assume
sin q
cos q - sin q
PX = >
cos q H8 1 2B
x x T
sin q

=>
cos q - sin q x1
sin q cos q H>x2H . in
=>
x1 cos q - x2 sin q
x1 sin q + x2 cos qH . co
i a
PX =
= 2
x +x
o d
(x1 cos q - x2 sin q) 2 + (x1 sin q + x2 cos q) 2
2

.n
1 2

PX = X

w
Hence (B) is correct option.

w
Q. 39 Let x (t) = rect^t - 12 h (where rect (x) = 1 for - 12 # x # 12 and zero otherwise. If
sin (px)
sinc (x) = px , then the Fourier Transform of x (t) + x (- t) will be given by

w
(A) sinc` w j
2p
(B) 2 sinc` w j
2p

Sol. 39
© 2p
Given signal
w
(C) 2 sinc` j cos ` j w
2
(D) sinc` j sin ` w j
w
2p 2

x (t) = rect `t - 1 j
2
1, - # t - 1 # 1 or 0 # t # 1
1
So, x (t) = * 2 2 2
0, elsewhere
Similarly
x (- t) = rect`- t - 1 j
2
1, - # - t - 1 # 1 or - 1 # t # 0
1
x (- t) = * 2 2 2
0, elsewhere

#- 3 x (t) e- jwt dt + #- 3 x (- t) e- jwt dt


3 3
F [x (t) + x (- t)] =
1 0
= #0 (1) e- jwt dt + #- 1 (1) e- jwt dt
GATE SOLVED PAPER - EE 2008

- jw t 1
- jw t 0
= ;e E +;e E
- jw 0 - jw - 1

= 1 (1 - e - jw) + 1 (e jw - 1)
jw jw
- j w /2 j w /2
=e (e jw/2 - e - jw/2) + e (e jw/2 - e - jw/2)
jw jw
(e jw/2 - e - jw/2) (e - jw/2 + e jw/2)
=
jw

= 2 sin ` w j $ 2 cos ` w j
w 2 2
w
= 2 cos sinc` j w
2 2p
Hence (C) is correct option.

. in
co
Q. 40 Two perfectly matched silicon transistor are connected as shown in the figure
assuming the b of the transistors to be very high and the forward voltage drop

.
in diodes to be 0.7 V, the value of current I is

a
d i
n o
.
w
w
(A) 0 mA
(B) 3.6 mA

w
(C) 4.3 mA

©
(D) 5.7 mA
Sol. 40 Hence (C) is correct option.

This is a current mirror circuit. Since b is high so IC1 = IC2, IB1 = IB2
VB = (- 5 + 0.7)
=- 4.3 volt
Diode D1 is forward biased.
So, current I is,
I = IC2 = IC1
0 - (- 4.3)
= = 4.3 mA
1
GATE SOLVED PAPER - EE 2008

Q. 41 In the voltage doubler circuit shown in the figure, the switch ‘S ’ is closed at t = 0
. Assuming diodes D1 and D2 to be ideal, load resistance to be infinite and initial
capacitor voltages to be zero. The steady state voltage across capacitor C1 and
C2 will be

(A) Vc1 = 10 V,Vc2 = 5 V


(B) Vc1 = 10 V,Vc2 =- 5 V
(C) Vc1 = 5 V,Vc2 = 10 V
(D) Vc1 = 5 V,Vc2 =- 10 V
Sol. 41 In positive half cycle of input, diode D1 is in forward bias and D2 is off, the

in
equivalent circuit is

.
. co
i a
o d
Capacitor C1 will charge upto + 5 volt. VC1 =+ 5 volt

.n
In negative halt cycle diode D1 is off and D2 is on.

w
w
w
©
Now capacitor VC2 will charge upto - 10 volt in opposite direction.
Hence (D) is correct option

Q. 42 The block diagrams of two of half wave rectifiers are shown in the figure. The
transfer characteristics of the rectifiers are also shown within the block.

It is desired to make full wave rectifier using above two half-wave rectifiers. The
resultants circuit will be
GATE SOLVED PAPER - EE 2008

. in
. co
i a
o d
. n
w
w
w
Sol. 42
©
Let input Vin is a sine wave shown below

According to given transfer characteristics of rectifiers output of rectifier P is.

Similarly output of rectifier Q is


GATE SOLVED PAPER - EE 2008

Output of a full wave rectifier is given as

To get output V0
V0 = K (- VP + VQ) K - gain of op-amp

in
So, P should connected at inverting terminal of op-amp and Q with non-inverting

.
terminal

co
Hence (D) is correct Option
A 3-line to 8-line decoder, with active low outputs, is used to implement a

.
Q. 43
3-variable Boolean function as shown in the figure

i a
o d
.n
w
w
The simplified form of Boolean function F (A, B, C) implemented in ‘Product of
Sum’ form will be

w
(A) (X + Z) (X + Y + Z ) (Y + Z)
(B) (X + Z ) (X + Y + Z) (Y + Z )

©
(C) (X + Y + Z) (X + Y + Z) (X + Y + Z) (X + Y + Z )
(D) (X + Y + Z) (X + Y + Z ) (X + Y + Z) (X + Y + Z )
Sol. 43 In SOP form, F is written as
F = Sm (1, 3, 5, 6)
= X Y Z + X YZ + XY Z + XYZ
Solving from K- map

F = X Z + Y Z + XYZ
In POS form
F = (Y + Z) (X + Z) (X + Y + Z )
since all outputs are active low so each input in above expression is
GATE SOLVED PAPER - EE 2008

complemented
F = (Y + Z ) (X + Z ) (X + Y + Z)
Hence (B) is correct option.
Q. 44 The truth table of monoshot shown in the figure is given in the table below :

Two monoshots, one positive edge triggered and other negative edge triggered,
are connected shown in the figure, The pulse widths of the two monoshot outputs
Q1 and Q2 are TON and TON respectively.

in
1 2

.
.co
i a
o d
. n
The frequency and the duty cycle of the signal at Q1 will respectively be
(A) f = 1 , D= 1

w
TON + TON
1 2
5
TON

w
(B) f = 1 , D= 2

TON + TON TON + TON

w
1 2 1 2

TON
(C) f = 1 , D = 1

TON TON + TON

©
1 1 2

TON
(D) f = 1 , D = 1

TON2
TON + TON
1 2

Sol. 44 In this case


f = 1
TON1 + TON 2
TON 2
and, D =
TON1 + TON 2
Hence (B) is correct option.

Q. 45 The content of some of the memory location in an 8085 accumulator based system
are given below
Address Content
g g
26FE 00
26FF 01
2700 02
2701 03
2702 04
g g
GATE SOLVED PAPER - EE 2008

The content of stack (SP), program counter (PC) and (H,L) are 2700 H, 2100 H
and 0000 H respectively. When the following sequence of instruction are executed.
2100 H: DAD SP
2101 H: PCHL
the content of (SP) and (PC) at the end of execution will be
(A) PC = 2102 H, SP = 2700 H (B) PC = 2700 H, SP = 2700 H
(C) PC = 2800 H, SP = 26FE H (D) PC = 2A02 H, SP = 2702 H
Sol. 45 Given that
SP = 2700 H
PC = 2100 H
HL = 0000 H
Executing given instruction set in following steps,
DAD SP &Add register pair (SP) to HL register
HL = HL + SP

in
HL = 0000 H + 2700 H

.
HL = 2700 H

co
PCHL & Load program counter with HL contents

.
PC = HL = 2700 H

a
So after execution contents are,

i
PC = 2700 H, HL = 2700 H

d
Hence (B) is correct option.

o
.n
Q. 46 A waveform generator circuit using OPAMPs is shown in the figure. It produces
a triangular wave at point ‘P’ with a peak to peak voltage of 5 V for Vi = 0 V .

w
w
w
©
If the voltage Vi is made + 2.5 V, the voltage waveform at point ‘P’ will become
GATE SOLVED PAPER - EE 2008

Sol. 46 Hence ( ) is correct option.


Q. 47 A 230 V, 50 Hz, 4-pole, single-phase induction motor is rotating in the clockwise
(forward) direction at a speed of 1425 rpm. If the rotor resistance at standstill is
7.8 W, then the effective rotor resistance in the backward branch of the equivalent
circuit will be
(A) 2 W (B) 4 W

in
(C) 78 W (D) 156 W

.
co
Sol. 47 Given: 230 V, 50 Hz, 4-Pole, 1-f induction motor is rotating in
clock-wise(forward) direction
Ns = 1425 rpm

a .
i
Rotar resistance at stand still(R2 ) = 7.8 W

d
So

o
Ns = 120 # 50 = 1500
4

.
Slip(S ) =
n
1500
1500
- 1425 = 0.05

w
Resistance in backward branch rb = R2

w
2-S
= 7 . 8

w
2 - 0.05
=4W

©
Hence (B) is correct option.
Q. 48 A 400 V, 50 Hz 30 hp, three-phase induction motor is drawing
50 A current at 0.8 power factor lagging. The stator and rotor copper losses are
1.5 kW and 900 W respectively. The friction and windage losses are 1050 W and
the core losses are 1200 W. The air-gap power of the motor will be
(A) 23.06 kW (B) 24.11 kW
(C) 25.01 kW (D) 26.21 kW
Sol. 48 Given: a 400 V, 50 Hz, 30 hp, 3-f induction motor
Current = 50 A at 0.8 p.f. lagging
Stator and rotor copper losses are 1.5 kW and 900 W
fraction and windage losses = 1050 W
Core losses = 1200 W = 1.2 kW
So,
Input power in stator = 3 # 400 # 50 # 0.8
= 27.71 kW
Air gap power = 27.71 - 1.5 - 1.2
= 25.01 kW
Hence (C) is correct option.
GATE SOLVED PAPER - EE 2008

Q. 49 The core of a two-winding transformer is subjected to a magnetic flux variation


as indicated in the figure.

The induced emf (ers) in the secondary winding as a function of time will be of
the form

. in
. co
i a
o d
.n
w
w
Sol. 49 Hence (A) is correct option.
df

w
Induced emf in secondary =- N2
dt

©
During -0 < t < 1
df
E1 =- (100) =- 12 V
dt
E1 and E2 are in opposition
E2 = 2E1 = 24 V

During time 1 < t < 2


df
= 0 , then E1 = E2 = 0
dt
During 2 < t < 2.5
df
E1 =- (100) =- 24 V
dt
Then
E2 =- 0 - 48 V
GATE SOLVED PAPER - EE 2008

Q. 50 Voltages phasors at the two terminals of a transmission line of length 70 km have


a magnitude of 1.0 per unit but are 180 degree out of phase. Assuming that the
maximum load current in the line is 1/5th
of minimum 3-phase fault current. Which one of the following transmission line
protection schemes will not pick up for this condition ?
(A) Distance protection using ohm relay with zoen-1 set to 80% of the line
impedance.
(B) Directional over current protection set to pick up at 1.25 times the
maximum load current
(C) Pilot relaying system with directional comparison scheme
(D) Pilot relaying system with segregated phase comparison scheme
Sol. 50 Hence ( ) is Correct Option

in
Q. 51 A loss less transmission line having Surge Impedance Loading (SIL) of 2280 MW

.
is provided with a uniformly distributed series capacitive compensation of 30%.

co
Then, SIL of the compensated transmission line will be
(A) 1835 MW (B) 2280 MW

a.
(C) 2725 MW (D) 3257 MW
Sol. 51 SIL has no effect of compensation
SIL = 2280 MW
d i
o
So

n
Hence (B) is correct option.
Q. 52
.
A loss less power system has to serve a load of 250 MW. There are tow generation

w
(G 1 and G 2 ) in the system with cost curves C1 and C2 respectively defined as

w
follows ;
2
C1 (PG1) = PG1 + 0.055 # PG1

w
2
C2 (PG2) = 3PG2 + 0.03 # PG2
Where PG1 and PG2 are the MW injections from generator G 1 and G 2 respectively.

©
Thus, the minimum cost dispatch will be
(A) PG1 = 250 MW; PG2 = 0 MW
(B) PG1 = 150 MW; PG2 = 100 MW
(C) PG1 = 100 MW; PG2 = 150 MW
(D) PG1 = 0 MW; PG2 = 250 MW
Sol. 52 Hence (C) is correct option
Given PG1 + PG2 = 250 MW ...(1)
C1 (PG1) = PG1 + 0.055PG12
and 4 ...(2)
C2 (PG2) = 3PG2 + 0.03PG22
from equation (2)
dC1 = 1 + 0.11PG1 ...(3a)
dPG1
and dC2 = 3 + 0.06PG2 ...(3b)
dPG2
Since the system is loss-less
Therefore dC1 = dC2
dPG1 dPG2

So from equations (3a) and (3b)


GATE SOLVED PAPER - EE 2008

We have 0.11PG1 - 0.06PG2 = 2 ...(4)


Now solving equation (1) and (4), we get
PG1 = 100 MW
PG2 = 150 MW

Q. 53 A loss less single machine infinite bus power system is shown below :

The synchronous generator transfers 1.0 per unit of power to the infinite bus. The
critical clearing time of circuit breaker is 0.28 s. If another identical synchronous
generator is connected in parallel to the existing generator and each generator is
scheduled to supply 0.5 per unit of power, then the critical clearing time of the
circuit breaker will

in
(A) reduce to 0.14 s (B) reduce but will be more than 0.14

.
s

co
(C) remain constant at 0.28 s (D) increase beyond 0.28 s

.
Sol. 53 After connecting both the generators in parallel and scheduled to supply 0.5 Pu

i a
of power results the increase in the current.

d
` Critical clearing time will reduced from 0.28 s but will not be less than 0.14 s

o
for transient stability purpose.

.n
Hence (B) is correct option.
Q. 54 Single line diagram of a 4-bus single source distribution system is shown below.

w
Branches e1, e2, e3 and e4 have equal impedances. The load current values indicated

w
in the figure are in per unit.

w
©

Distribution company’s policy requires radial system operation with minimum


loss. This can be achieved by opening of the branch
(A) e1 (B) e2
(C) e3 (D) e4
Sol. 54 Given that the each section has equal impedance.
Let it be R or Z , then by using the formula
line losses = / I2 R
GATE SOLVED PAPER - EE 2008

On removing (e1); losses = (1) 2 R + (1 + 2) 2 R + (1 + 2 + 5) 2 R


= R + 9R + 64R = 74R
Similarly,
On removing e2 ;losses = 52 R + (5 + 2) 2 R + (5 + 2 + 1) 2 R
= 138R
lossess on removing e 3 = (1) 2 R + (2) 2 R + (5 + 2) 2 R
= 1R + 4R + 49R
= 54R
on removing e 4 lossless = (2) 2 R + (2 + 1) 2 R + 52 R
= 4R + 9R + 25R
= 38R
So, minimum losses are gained by removing e 4 branch.

in
Hence (D) is correct option

.
co
Q. 55 A single phase fully controlled bridge converter supplies a load drawing constant
and ripple free load current, if the triggering angle is 30c, the input power factor

.
will be

a
(A) 0.65 (B) 0.78
(C) 0.85

d i (D) 0.866

o
Sol. 55 Given a = 30c, in a 1-f fully bridge converter

n
we know that,

. Power factor = Distortion factor # cos a

w
D.f. (Distortion factor) = Is(fundamental) /Is = 0.9

w power factor = 0.9 # cos 30c

w
= 0.78
Hence (B) is correct option
Q. 56
©
A single-phase half controlled converter shown in the figure feeding power to
highly inductive load. The converter is operating at a firing angle of 60c.

If the firing pulses are suddenly removed, the steady state voltage (V0) waveform
of the converter will become
GATE SOLVED PAPER - EE 2008

in
Sol. 56 Output of this

.
. co
i a
o d
.n
Here the inductor makes T1 and T3 in ON because current passing through T1

w
and T3 is more than the holding current.
Hence (A) is correct option
Q. 57
w
A 220 V 20 A, 1000 rpm, separately excited dc motor has an armature resistance of

w
2.5 W. The motor is controlled by a step down chopper with a frequency of 1 kHz

©
. The input dc voltage to the chopper is 250 V. The duty cycle of the chopper for
the motor to operate at a speed of 600 rpm delivering the rated torque will be
(A) 0.518 (B) 0.608
(C) 0.852 (D) 0.902
Sol. 57 Hence ( ) is Correct Option

Q. 58 A 220 V, 1400 rpm, 40 A separately excited dc motor has an armature resistance


of 0.4 W. The motor is fed from a single phase circulating current dual converter
with an input ac line voltage of 220 V (rms). The approximate firing angles of
the dual converter for motoring operating at 50% of rated torque and 1000 rpm
will be
(A) 43c, 137c (B) 43c, 47c
(C) 39c, 141c (D) 39c, 51c
Sol. 58 Hence ( ) is Correct Option
GATE SOLVED PAPER - EE 2008

Q. 59 A single phase source inverter is feeding a purely inductive load as shown in the
figure
The inverter is operated at 50 Hz in 180c square wave mode. Assume that the
load current does not have any dc component. The peak value of the inductor
current i0 will be

(A) 6.37 A (B) 10 A


(C) 20 A (D) 40 A

. in
Sol. 59 Input is given as

. co
i a
o d
. n
Here load current does not have any dc component

w
`Peak current occur at (p/w)
Vs = L di

w
`
dt
200 = 0.1 # di

w
dt
p
di = a kb 1 l = 1

©
Here
2p 50 100
So di(max) = 200 # 1 1 = 20 A
100 # 0.1
Hence (C) is correct option
Q. 60 A 400 V, 50 Hz, 4-pole, 1400 rpm, star connected squirrel cage induction motor
has the following parameters referred to the stator:
R'r = 1.0 W, Xs = X'r = 1.5 W
Neglect stator resistance and core and rotational losses of the motor. The motor
is controlled from a 3-phase voltage source inverter with constant V/f control.
The stator line-to-line voltage(rms) and frequency to obtain the maximum torque
at starting will be :
(A) 20.6 V, 2.7 Hz (B) 133.3 V, 16.7 Hz
(C) 266.6 V, 33.3 Hz (D) 323.3 V, 40.3 Hz
Sol. 60 Given 400 V, 50 Hz, 4-Pole, 1400 rpm star connected squirrel cage induction
motor.
R = 1.00 W, Xs = Xlr = 1.5 W
So,
for max. torque slip
Sm = Rlr
Xsm + Xlrm
GATE SOLVED PAPER - EE 2008

for starting torque Sm = 1


Then
Xsm + Xlrm = Rlr
2pfm Ls + 0.2pfm Llr = 1
Frequency at max. torque
fm = 1
2p (Ls + Llr )
Ls = Xs = 1.5
2p # 50 2p # 50
Llr = 1.5
2p # 50
fm = 1 = 50
1.5 + 1.5 3
50 50
fm = 16.7 Hz
In const V/f control method
V1 = 400 = 8
f1 50
a V2
f1
=8

. in
co
So

.
V2 = f2 # 8

a
= 16.7 # 8
V2 = 133.3 V
Hence (B) is correct option.
d i
o
.n
Q. 61 A single phase fully controlled converter bridge is used for electrical braking of a
separately excited dc motor. The dc motor load is represented by an equivalent

w
circuit as shown in the figure.

w
w
©
Assume that the load inductance is sufficient to ensure continuous and ripple free
load current. The firing angle of the bridge for a load current of I0 = 10 A will be
(A) 44c (B) 51c
(C) 129c (D) 136c
Sol. 61 Here for continuous conduction mode, by Kirchoff’s voltage law, average load
current

V - 2Ia + 150 = 0
GATE SOLVED PAPER - EE 2008

Ia = V + 150
2
` I1 = 10 A, So
V =- 130 V
2Vm cos a =- 130
p
2 # 2 # 230 cos a =- 130c
p
a = 129c
Hence (C) is correct option.
Q. 62 A three phase fully controlled bridge converter is feeding a load drawing a constant
and ripple free load current of 10 A at a firing angle of 30c. The approximate Total
harmonic Distortion (%THD) and the rms value of fundamental component of
input current will respectively be

in
(A) 31% and 6.8 A (B) 31% and 7.8 A
(C) 66% and 6.8 A
.
(D) 66% and 7.8 A

co
Sol. 62 Hence (B) is correct option.
Total rms current Ia =

a .2 10 = 8.16 A

i
3#

d
Fundamental current Ia1 = 0.78 # 10 = 7.8 A

n o THD = 1 -1

.
DF2
where

w DF = Ia1 = 0.78 # 10 = 0.955


Ia 0.816 # 10

w THD = b 1 l - 1 = 31%
2

w
`
0.955

©
Q. 63 In the circuit shown in the figure, the switch is operated at a duty cycle of 0.5.
A large capacitor is connected across the load. The inductor current is assumed
to be continuous.

The average voltage across the load and the average current through the diode
will respectively be
(A) 10 V, 2 A (B) 10 V, 8 A
(C) 40 V 2 A (D) 40 V, 8 A
Sol. 63 Hence (C) is correct option.
GATE SOLVED PAPER - EE 2008

In the given diagram


when switch S is open I 0 = IL = 4 A, Vs = 20 V
when switch S is closed ID = 0, V0 = 0 V
Duty cycle = 0.5 so average voltage is Vs
1-d
Average current = 0 + 4 = 2 amp
2
Average voltage = 20 = 40 V
1 - 0.5
Q. 64 The transfer function of a linear time invariant system is given as
G (s) = 2 1
s + 3s + 2
The steady state value of the output of the system for a unit impulse input
applied at time instant t = 1 will be
(A) 0 (B) 0.5
(C) 1 (D) 2

in
Sol. 64 Given transfer function

.
G (s) = 1
s 2 + 3s + 2

co
Input r (t) = d (t - 1)

.
R (s) = L [d (t - 1)] = e- s

a
Output is given by
Y (s) = R (s) G (s)
Y (s) = 2 e
-s

d i
o
s + 3s + 2
Steady state value of output

.n
lim y (t) = lim sY (s)
t"3 s"0

w
se- s
= lim =0
2
s + 3s + 2

w
s"0

Hence (A) is correct option.


Q. 65
w
The transfer functions of two compensators are given below :

©
10 (s + 1)
C1 = , C2 = s + 10
(s + 10) 10 (s + 1)
Which one of the following statements is correct ?
(A) C1 is lead compensator and C2 is a lag compensator
(B) C1 is a lag compensator and C2 is a lead compensator
(C) Both C1 and C2 are lead compensator
(D) Both C1 and C2 are lag compensator
Sol. 65 For C1 Phase is given by
qC = tan- 1 (w) - tan- 1 a w k
1
10
Jw - w N
qC1 = tan- 1 K 10 O
KK w2
O
1+ O
L 10 P
qC1 = tan c 9w
10 + w2 m
-1
> 0 (Phase lead)

Similarly for C2 , phase is


GATE SOLVED PAPER - EE 2008

qC2 = tan- 1 a w k - tan- 1 (w)


10
J w - wN
= tan K 10 2 O
-1
KK O
1+ w O
L 10 P
= tan c - 9w
10 + w2 m
-1
< 0 (Phase lag)

Hence (A) is correct option


Q. 66 The asymptotic Bode magnitude plot of a minimum phase transfer function is
shown in the figure :

. in
. co
i a
This transfer function has

o d
n
(A) Three poles and one zero (B) Two poles and one zero

.
(C) Two poles and two zero (D) One pole and two zeros
Sol. 66
w
From the given bode plot we can analyze that:

w
Slope - 40 dB/decade"2 poles

w
Slope - 20 dB/decade (Slope changes by + 20 dB/decade)"1 Zero
Slope 0 dB/decade (Slope changes by + 20 dB/decade)"1 Zero

©
So there are 2 poles and 2 zeroes in the transfer function.
Hence (C) is correct option.
Q. 67 Figure shows a feedback system where K > 0

The range of K for which the system is stable will be given by


(A) 0 < K < 30 (B) 0 < K < 39
(C) 0 < K < 390 (D) K > 390
Sol. 67 Characteristic equation for the system
1+ K =0
s (s + 3) (s + 10)

s (s + 3) (s + 10) + K = 0
s3 + 13s2 + 30s + K = 0

Applying Routh’s stability criteria


GATE SOLVED PAPER - EE 2008

s3 1 30
s2 13 K
s1 (13 # 30) - K
13
s0 K
For stability there should be no sign change in first column
So, 390 - K > 0 & K < 390
K >0
0 < K < 90
Hence (C) is correct option
Q. 68 The transfer function of a system is given as
100
s2 + 20s + 100
The system is

in
(A) An over damped system (B) An under damped system

.
(C) A critically damped system (D) An unstable system

co
Sol. 68 Given transfer function is

.
H (s)) = 100

a
s2 + 20s + 100

i
Characteristic equation of the system is given by

d
s2 + 20s + 100 = 0

o
wn2 = 100 & wn = 10 rad/sec.

.n
2xwn = 20
or x = 20 = 1

w
2 # 10
(x = 1) so system is critically damped.

w
Hence (C) is correct option.
Q. 69
w
Two sinusoidal signals p (w1, t) = A sin w1 t and q (w2 t) are applied to X and Y

©
inputs of a dual channel CRO. The Lissajous figure displayed on the screen shown
below :
The signal q (w2 t) will be represented as

(A) q (w2 t) = A sin w2 t, w2 = 2w1 (B) q (w2 t) = A sin w2 t, w2 = w1 /2


(C) q (w2 t) = A cos w2 t, w2 = 2w1 (D) q (w2 t) = A cos w2 t, w2 = w1 /2
Sol. 69 Hence (D) is correct option.
GATE SOLVED PAPER - EE 2008

fY meeting points of horizontal tangents


Frequency ratio =
fX meeting points of vertical tangents
fY
=2
fX 4
1
fY = (fX )
2

in
w2 = w1 /2

.
Since the Lissajous figures are ellipse, so there is a phase difference of 90c exists

co
between vertical and horizontal inputs.

.
So q (w2 t) = A cos w2 t, w2 = w1 /2

Q. 70

i a
The ac bridge shown in the figure is used to measure the impedance Z .

o d
. n
w
w
w
If the bridge is balanced for oscillator frequency f = 2 kHz, then the impedance

©
Z will be
(A) (260 + j0) W (B) (0 + j200) W
(C) (260 - j200) W (D) (260 + j200) W
Sol. 70 Impedance of different branches is given as
ZAB = 500 W

ZBC = 1 + 300 W
j # 2p # 2 # 103 # 0.398 mF
- (- 200j + 300) W
ZAD = j # 2p # 2 # 103 # 15.91 mH + 300 W
- (200j + 300) W
To balance the bridge
ZAB ZCD = ZAD ZBC
500Z = (200j + 300) (- 200j + 300)
500Z = 130000
Z = (260 + j0) W
Hence (A) is correct option.
GATE SOLVED PAPER - EE 2008

Common Data for Questions 71, 72 and 73:


Consider a power system shown below:

Given that: Vs1 = Vs2 = 1 + j0 p.u ;


The positive sequence impedance are
Zs1 = Zs2 = 0.001 + j0.01 p.u and ZL = 0.006 + j0.06 p.u
3-phase Base MVA = 100
voltage base = 400 kV(Line to Line)
Nominal system frequency = 50 Hz.
The reference voltage for phase ‘a’ is defined as V (t) = Vm cos (wt).
A symmetrical three phase fault occurs at centre of the line, i.e. point ‘F’ at
time ‘t 0 ’. The positive sequence impedance from source S1 to point ‘F’ equals

in
0.004 + j0.04 p.u. The wave form corresponding to phase ‘a’ fault current from

.
bus X reveals that decaying d.c. offset current is negative and in magnitude at its

co
maximum initial value, Assume that the negative sequence impedances are equal

.
to postive sequence impedance and the zero sequence impedances are three times

a
positive sequence impedances.
Q. 71

d i
The instant (t0) of the fault will be

o
(A) 4.682 ms (B) 9.667 ms

.n
(C) 14.667 ms (D) 19.667 ms
Sol. 71 Given

w
V (t) = Vm cos (wt)

w
For symmetrical 3 - f fault, current after the fault
i (t) = Ae- (R/L) t + 2 Vm cos (wt - a)
w Z
At the instant of fault i.e t = t 0 , the total current i (t) = 0

© ` 0 = Ae- (R/L) t + 2 Vm cos (wt 0 - a)


0

Z
Ae- (R/L) t =- 2 Vm cos (wt 0 - a)
0

Z
Maximum value of the dc offset current
Ae- (R/L) t =- 2 Vm cos (wt 0 - a)
0

Z
For this to be negative max.
(wt 0 - a) = 0
or t0 = a ...(1)
w
and Z = 0.004 + j0.04
Z = Z +a = 0.0401995+84.29c
` a = 84.29cor 1.471 rad.
From equation (1)
t 0 = 1.471 = 0.00468 sec
(2p # 50)
t 0 = 4.682 ms
Hence (A) is correct option.
GATE SOLVED PAPER - EE 2008

Q. 72 The rms value of the component of fault current (If ) will be


(A) 3.59 kA (B) 5.07 kA
(C) 7.18 kA (D) 10.15 kA
Sol. 72

Since the fault ‘F’ is at mid point of the system, therefore impedance seen is same
from both sides.

Z = 0.0201+84.29c
2

in
Z1 (Positive sequence) = Z = 0.0201+84.29c

.
2

co
also Z1 = Z2 = Z 0 (for 3-f fault)

` I f (pu) = 1+0c =
.
1+0c

a
Z1 0.0201+84.29c
So magnitude If
i
= 49.8

od
(p.u.)

` Fault current I f = 49.8 # 100

.n
3 # 400
= 7.18 kA

w
Hence (C) is correct option.

w
Q. 73 Instead of the three phase fault, if a single line to ground fault occurs on phase

w
‘a’ at point ‘F’ with zero fault impedance, then the rms of the ac component of
fault current (Ix) for phase ‘a’ will be

©
(A) 4.97 p.u (B) 7.0 p.u
(C) 14.93 p.u (D) 29.85 p.u
Sol. 73 If fault is LG in phase ‘a’

Z1 = Z = 0.0201+84.29c
2
Z2 = Z1 = 0.0201+84.29c
and Z 0 = 3Z1 = 0.0603+84.29c

Then
Ia /3 = Ia1 = Ia2 = Ia0
GATE SOLVED PAPER - EE 2008

` Ia1 (pu) = 1.0+0c


Z1 + Z 2 + Z 0
and Ia1 = 1.0 = 9.95 pu
(0.0201 + 0.0201 + 0.0603)
Fault Current I f = Ia = 3Ia1 = 29.85 pu

So Fault current I f = 29.85 # 100


3 # 400
= 4.97 kA
Hence (A) is correct option.

Common Data for Questions 74 and 75:


A 3-phase, 440 V, 50 Hz, 4-pole slip ring induction motor is feed from the rotor side
through an auto-transformer and the stator is connected to a variable resistance
as shown in the figure.

. in
. co
i a
o d
.n
w
The motor is coupled to a 220 V, separately excited d.c generator feeding power

w
to fixed resistance of 10 W. Two-wattmeter method is used to measure the input

w
power to induction motor. The variable resistance is adjusted such the motor runs
at 1410 rpm and the following readings were recorded W1 = 1800 W, W2 =- 200

©
W.
Q. 74 The speed of rotation of stator magnetic field with respect to rotor structure will
be
(A) 90 rpm in the direction of rotation
(B) 90 rpm in the opposite direction of rotation
(C) 1500 rpm in the direction of rotation
(D) 1500 rpm in the opposite direction of rotation
Sol. 74 Given 3-f, 440 V, 50 Hz, 4-Pole slip ring motor
Motor is coupled to 220 V
N = 1410 rpm, W1 = 1800 W, W2 = 200 W
So,
120f
Ns =
P
= 120 # 50 = 1500 rpm
4
Relative speed = 1500 - 1410
= 90 rpm in the direction of rotation.
GATE SOLVED PAPER - EE 2008

Hence (A) is correct option.


Q. 75 Neglecting all losses of both the machines, the dc generator power output and the
current through resistance (Rex) will respectively be
(A) 96 W, 3.10 A (B) 120 W, 3.46 A
(C) 1504 W, 12.26 A (D) 1880 W, 13.71 A
Sol. 75 Neglecting losses of both machines
Slip(S ) = Ns - N
Ns
= 1500 - 1410 = 0.06
1500
total power input to induction motor is
Pin = 1800 - 200 = 1600 W
Output power of induction motor

in
Pout = (1 - S) Pin

.
= (1 - 0.06) 1600

co
= 1504 W
Losses are neglected so dc generator input power = output power

a .
i
= 1504 W

d
So,

o
I2 R = 1504

n
1504 = 12.26 A

.
I =
10

w
Hence (C) is correct option.

w
w
Linked Answer Questions: Q.76 to Q.85 carry two marks each.

©
Statement for Linked Answer Questions 76 and 77:
The current i (t) sketched in the figure flows through a initially uncharged 0.3 nF
capacitor.

Q. 76 The charge stored in the capacitor at t = 5 ms , will be


(A) 8 nC (B) 10 nC
(C) 13 nC (D) 16 nC
GATE SOLVED PAPER - EE 2008

Sol. 76 Charge stored at t = 5 m sec


5
Q = # i (t) dt
0

=area under the curve

Q =Area OABCDO

. in
co
=Area (OAD)+Area(AEB)+Area(EBCD)
= 1#2#4+1#2#3+3#2

.
2 2
= 4+3+6
ia
d
= 13 nC

o
Hence (C) is correct option

.n
Q. 77 The capacitor charged upto 5 ms, as per the current profile given in the figure,

w
is connected across an inductor of 0.6 mH. Then the value of voltage across the
capacitor after 1 ms will approximately be
(A) 18.8 V
w (B) 23.5 V

w
(C) - 23.5 V (D) - 30.6 V

©
Sol. 77 Initial voltage across capacitor
Q
V0 = o = 13 nC
C 0.3 nF
= 43.33 Volt
When capacitor is connected across an inductor it will give sinusoidal esponse
as
vc (t) = Vo cos wo t
where wo = 1
LC
= 1
0.3 # 10- 9 # 0.6 # 10- 3
= 2.35 # 106 rad/sec
at t = 1 m sec
So, vc (t) = 43.33 cos (2.35 # 106 # 1 # 10- 6)
= 43.33 # (- 0.70)
=- 30.44 V
Hence (D) is correct option.
GATE SOLVED PAPER - EE 2008

Statement for Linked Answer Question 78 and 79.


The state space equation of a system is described by Xo = AX + Bu,Y = CX
where X is state vector, u is input, Y is output and
0 1 0
A == G, B = = G, C = [1 0]
0 -2 1
Q. 78 The transfer function G(s) of this system will be
(A) s (B) s + 1
(s + 2) s (s - 2)
(C) s (D) 1
(s - 2) s (s + 2)
Sol. 78 State space equation of the system is given by,
Xo = AX + Bu
Y = CX
Taking Laplace transform on both sides of the equations.

in
sX (s) = AX (s) + BU (s)
(sI - A) X (s) = BU (s)
.
co
X (s) = (sI - A) - 1 BU (s)

.
` Y (s) = CX (s)

a
So Y (s) = C (sI - A) - 1 BU (s)

T.F =
Y (s)
U (s)
d i
= C (sI - A) - 1 B

n o
s 0
(sI - A) = > H - >
0 1 s -1
0 - 2H >0 s + 2H
=

.
0 s
1 >s + 2 1H

w
(sI - A) - 1 =
s (s + 2) 0 s

w
R V
S1 1 W
s s (s + 2)W

w
= SS 1 W
0
S (s + 2) W

©
Transfer function T X
R V R V
S1 1 W S 1 W
s s (s + 2)W 0 Ss (s + 2)W
G (s) = C [sI - A] - 1 B = 81 0BSS 1 W>1H = 81 0BS 1 W
0 S (s + 2) W
S (s + 2) W
1 T X T X
=
s (s + 2)
Hence (D) is correct option.
Q. 79 A unity feedback is provided to the above system G (s) to make it a closed loop
system as shown in figure.

For a unit step input r (t), the steady state error in the input will be
(A) 0 (B) 1
(C) 2 (D) 3
Sol. 79 Steady state error is given by,
sR (s)
ess = lim = G
s " 0 1 + G (s) H (s)
GATE SOLVED PAPER - EE 2008

Here R (s) = L [r (t)] = 1 (Unit step input)


s

G (s) = 1
s (s + 2)
H (s) = 1 (Unity feed back)
R V
S sb 1 l W
s
So, ess = lim S W
s"0S
1 + 1 W
S s (s + 2) W
T X
s (s + 2)
= lim = G
s " 0 s (s + 2) + 1

=0
Hence (A) is correct option.

in
Statement for Linked Answer Questions 80 and 81.

.
A general filter circuit is shown in the figure :

. co
i a
o d
.n
w
Q. 80

w
If R1 = R2 = RA and R3 = R4 = RB , the circuit acts as a

w
(A) all pass filter (B) band pass filter
(C) high pass filter (D) low pass filter
Sol. 80
©
For low frequencies,
w " 0 , so 1 " 3
wC
Equivalent circuit is,

By applying node equation at positive and negative input terminals of op-amp.


vA - vi + vA - vo = 0
R1 R2
2vA = vi + vo ,
GATE SOLVED PAPER - EE 2008

a R1 = R 2 = R A
Similarly,
vA - vi + vA - 0 = 0
R3 R4

2vA = vin , a R 3 = R 4 = RB

So, vo = 0
It will stop low frequency signals.
For high frequencies,
w " 3 , then 1 " 0
wC
Equivalent circuit is,

. in
. co
i a
o d
Output,

.
vo = vi
n
w
So it will pass high frequency signal.
This is a high pass filter.

w
Hence (C) is correct option

w
Q. 81 The output of the filter in Q.80 is given to the circuit in figure :
The gain v/s frequency characteristic of the output (vo) will be

Sol. 81 In Q.80 cutoff frequency of high pass filter is given by,


wh = 1
2pRA C
Here given circuit is a low pass filter with cutoff frequency,
GATE SOLVED PAPER - EE 2008

wL = 1 = 2
2p RA
C 2 p RAC
2
wL = 2wh
When both the circuits are connected together, equivalent circuit is,

So this is is Band pass filter, amplitude response is given by.

Hence (D) is correct option.

Statement for Linked Answer Question 82 and 83.

in
Q. 82 A 240 V, dc shunt motor draws 15 A while supplying the rated load at a speed

.
of 80 rad/s. The armature resistance is 0.5 W and the field winding resistance is

co
80 W.

.
The net voltage across the armature resistance at the time of plugging will be

a
(A) 6 V (B) 234 V
(C) 240 V

d i (D) 474 V

o
Sol. 82 Given: V = 240 V , dc shunt motor

.n
I = 15 A
Rated load at a speed = 80 rad/s

wArmature Resistance = 0.5 W

w
Field winding Resistance = 80 W

w
So,
E = 240 - 12 # 0.5

©
E = 234
Vplugging = V + E
= 240 + 234
= 474 V
Hence (D) is correct option.
Q. 83 The external resistance to be added in the armature circuit to limit the armature
current to 125% of its rated value is
(A) 31.1 W (B) 31.9 W
(C) 15.1 W (D) 15.9 W]
Sol. 83

External Resistance to be added in the armature circuit to limit the armature


current to 125%.
So Ia = 12 # 1.25 = 474
Ra + R external
Ra + R external = 31.6
R external = 31.1 W
Hence (A) is correct option
GATE SOLVED PAPER - EE 2008

Statement for Linked Answer Question 84 and 85.


A synchronous motor is connected to an infinite bus at 1.0 pu voltage and draws
0.6 pu current at unity power factor. Its synchronous reactance is 1.0 pu resistance
is negligible.
Q. 84 The excitation voltage (E ) and load angle (d) will respectively be
(A) 0.8 pu and 36.86c lag (B) 0.8 pu and 36.86c lead
(C) 1.17 pu and 30.96c lead (D) 1.17 pu and 30.96c lag
Sol. 84 A synchronous motor is connected to an infinite bus at 1.0 p.u. voltage and
0.6 p.u. current at unity power factor. Reactance is 1.0 p.u. and resistance is
negligible.
So,
V = 1+0c p.u.

in
Ia = 0.6+0c p.u.

.
Zs = Ra + jXs = 0 + j1 = 1+90c p.u.

co
V = E+d + Ia Zs = 1+0c - 0.6+0c # 1+90c
E+d = 1.166+ - 30.96c p.u.

.
Excitation voltage = 1.17 p.u.

i a
Load angle (d) = 30.96c(lagging)

d
Hence (D) is correct option.

o
Q. 85 Keeping the excitation voltage same, the load on the motor is increased such that

n
the motor current increases by 20%. The operating power factor will become
(A) 0.995 lagging
. (B) 0.995 leading
(C) 0.791 lagging
w (D) 0.848 leading
Sol. 85

w
Hence (D) is correct option.

w
©
GATE SOLVED PAPER - EE 2008

Answer Sheet
1. (A) 19. (D) 37. (C) 55. (B) 73. (A)
2. (A) 20. (D) 38. (B) 56. (A) 74. (A)
3. (D) 21. (C) 39. (C) 57. (*) 75. (C)
4. (C) 22. (C) 40. (C) 58. (*) 76. (C)
5. (D) 23. (A) 41. (D) 59. (C) 77. (D)
6. (A) 24. (B) 42. (D) 60. (B) 78. (D)
7. (D) 25. (B) 43. (B) 61. (C) 79. (A)
8. (A) 26. (B) 44. (B) 62. (B) 80. (C)
9. (A) 27. (*) 45. (B) 63. (C) 81. (D)
10. (*) 28. (B) 46. (*) 64. (A) 82. (D)
11. (D) 29. (D) 47. (B) 65. (A) 83. (A)

in
12. (B) 30. (A) 48. (C) 66. (C) 84. (D)
13. (D) 31. (B) 49. (A)
.
67. (C) 85. (D)

co
14. (D) 32. (*) 50. (*) 68. (C)
15. (D) 33. (D) 51. (B)

a . 69. (D)

di
16. (B) 34. (B) 52. (C) 70. (A)
17. (A) 35. (A) 53. (B) 71. (A)
18. (D) 36. (D)

n o
54. (D) 72. (C)

.
**********

w
w
w
©

Anda mungkin juga menyukai